1answer.
Ask question
Login Signup
Ask question
All categories
  • English
  • Mathematics
  • Social Studies
  • Business
  • History
  • Health
  • Geography
  • Biology
  • Physics
  • Chemistry
  • Computers and Technology
  • Arts
  • World Languages
  • Spanish
  • French
  • German
  • Advanced Placement (AP)
  • SAT
  • Medicine
  • Law
  • Engineering
DanielleElmas [232]
3 years ago
7

Below are the range and standard deviation for a set of data. Use the range rule of thumb and compare it to the standard deviati

on listed below. Does the range rule of thumb produce an acceptable​ approximation? Suppose a researcher deems the approximation as acceptable if it has an error less than​ 15%. Range equals 2.76 standard deviation equals 0.807 the estimated standard deviation is nothing. ​(round to three decimal places as​ needed.) is this an acceptable​ approximation?
a. Yes​, because the error of the range rule of​ thumb's approximation is greater than​ 15%.
b. No​, because the error of the range rule of​ thumb's approximation is less than​ 15%.
c. Yes​, because the error of the range rule of​ thumb's approximation is less than​ 15%.
d. No​, because the error of the range rule of​ thumb's approximation is greater than​ 15%.
Mathematics
1 answer:
Liula [17]3 years ago
4 0

Solution: We are given:

Range =2.76

Standard-deviation=0.807

Now let's find the estimated value of standard deviation using the range rule of thumb. According to range rule of thumb, the estimate of standard deviation is:

S\approx \frac{2.76}{4}= 0.690

Therefore the estimated standard deviation is 0.690

is this an acceptable​ approximation?

Answer: c. Yes​, because the error of the range rule of​ thumb's approximation is less than​ 15%.

Explanation:

The difference between estimated standard deviation and actual standard deviation is:

0.807 - 0.690 = 0.117

Now let's find the percentage of error

\frac{0.117}{0.807} \times 100= 14.50\%.

Therefore, the error of the range rule of​ thumb's approximation is less than​ 15%.

You might be interested in
Write the equation of the line shown in point slope form (1,2) (3,6)
Kipish [7]

The point-slope form:

y-y_1=m(x-x_1)\\\\m=\dfrac{y_2-y_1}{x_2-x_1}

We have the points (1, 2) and (3, 6). Substitute:

m=\dfrac{6-2}{3-1}=\dfrac{4}{2}=2\\\\\boxed{y-2=2(x-1)}

7 0
3 years ago
The diagram shows a right-angled triangle.
Alexus [3.1K]

Step-by-step explanation:

there is no diagram so I'm just going to guess this however

sin x = 6/9

sin x = 2/3

sin x = 0.67

x = sin inverse of 0.67

x = 42.1

8 0
3 years ago
Help please and show your work
andrezito [222]
6: -8+4V
8: -9-15V OR -9+(-)15V
10:-17+33V
8 0
3 years ago
Staples sell for $60 a box .How many boxes can be bought for 3.45
kakasveta [241]

Answer:0.0575

Step-by-step explanation:

1 box cost $60

x boxes can be bought for $3.45

x=3.45/60

x=0.0575

4 0
3 years ago
In simplest radical form, what are the solutions to the quadratic equation 6 = x2 – 10x?
vichka [17]

Answer:

x = 5+\sqrt{31}\,\, and\,\, x=5-\sqrt{31}

Step-by-step explanation:

We need to solve the quadratic equation

6 = x^2 -10x

Rearranging we get,

x^2-10x-6=0

Using quadratic formula to solve the quadratic equation

x=\frac{-b\pm\sqrt{b^2-4ac}}{2a}

a= 1, b =-10 and c=6

Putting values in the quadratic formula

x=\frac{-(-10)\pm\sqrt{(-10)^2-4(1)(-6)}}{2(1)}\\x=\frac{10\pm\sqrt{100+24}}{2}\\x=\frac{10\pm\sqrt{124}}{2}\\x=\frac{10\pm\sqrt{2*2*31}}{2}\\x=\frac{10\pm\sqrt{2^2*31}}{2}\\x=\frac{10\pm2\sqrt{31}}{2}\\x = 5\pm\sqrt{31}

So, x=5+\sqrt{31}\,\, and\,\, x=5-\sqrt{31}

3 0
3 years ago
Read 2 more answers
Other questions:
  • The distance versus time plot for a particular object shows a quadratic relationship. Which column of distance data is possible
    6·1 answer
  • You can build a dog house in 6 hours. And you work 8 hours how long would it take you to build 10 dog house.
    15·2 answers
  • Determine the area under the standard normal curve that lies to the left of ​(a) z equals negative 1.53 commaz=−1.53, ​(b) z equ
    10·1 answer
  • The price of a computer component is decreasing at a rate of ​10% per year. State whether this decrease is linear or exponential
    6·1 answer
  • $70,000 - 17 1/2 % . <br>if i get this right i get the iphone xr​
    5·1 answer
  • Identify the slope and the y-intercept of 3x-4y=-16​
    8·2 answers
  • 4 sailboats to 12 motorboats
    7·1 answer
  • What is the translation of the point to its image? A (−1, 4)→ A' (3, 3)
    10·1 answer
  • The function f(x) = 2x 26 represents the distance a flock of birds travels in in miles. The function g(x) = x − 1 represents the
    8·1 answer
  • One possible integer of x for which 1/4 &lt; k/10 &lt; 1/3 is true?
    9·1 answer
Add answer
Login
Not registered? Fast signup
Signup
Login Signup
Ask question!